1

The expectation of the log of the absolute value of random variable with the standard normal distrbution can be computed based on the result provided here 1.

How can this be computed for an arbitrary normal distrbution. It is equivalent to finding the following expectation:

$$\mathbb E [\log|\alpha X + \beta|]=\int^{\infty}_{-\infty} \log(| \alpha x + \beta |) \frac{1}{\sqrt{2\pi}} e^{\frac{-x^2}{2} }{\rm d}x $$

where $X$ has the standard normal distribution.

For $\beta=0$, the formula is

$$\mathbb E [\log|\alpha X|]=\log|\alpha|-\frac{1}{2} (\gamma +\log (2))$$

where $\gamma$ denotes the Euler's constant.

Amir
  • 4,305
  • If we were to drop the absolute value from the logarithm then the result, along with various generalizations, is given here https://math.stackexchange.com/questions/1359584/an-integral-involving-a-gaussian-and-a-logarithm . – Przemo Jan 23 '24 at 15:17
  • What is your motivation for asking that question? Is it a pure mathematical curiosity or does it have real life applications? – Przemo Jan 23 '24 at 15:19
  • @Przemo I think it is a good measure for comparing the magnitude of different distributions; it is even finite for the Cauchy distribution: https://math.stackexchange.com/q/4838543/1231520.

    You may also see the background discussed here for another application: https://math.stackexchange.com/q/4843188/1231520. Note that $$\mathbb E \left( \log(|1-\alpha X^2|) \right )=\mathbb E \bigg [ \log(|1-\sqrt{\alpha}X|)+\log(|1+\sqrt{\alpha}X|) \bigg ]. $$

    – Amir Jan 23 '24 at 22:42
  • @Przemo Regarding your first comment above, after dropping the absolute value, the log cannot be used for negative values. How did you manage this? – Amir Jan 23 '24 at 22:51
  • Of course it can be. $\log (x) = \log(\left| x \right|) + \imath arg(x)$. – Przemo Jan 24 '24 at 10:14
  • @Przemo Nice! Now I see; you used the complex logarithm, and the real part of the formula provided in your question above (for the normal distribution) is equal to the solution here for $\beta=1$ and $\alpha=A$ . – Amir Jan 24 '24 at 10:48

1 Answers1

2

Notations

  • $\displaystyle F(z):=\frac{\sqrt{\pi}}{2}e^{-z^2}\text{erfi}(z)\text{ is the Dawson integral}$
  • $\gamma$ is the Eulero-Mascheroni constant
  • ${}_pF_q(a_1,...,a_p; b_1,...,b_q|z)$ is the hypergemetric function
  • P.V. is the principal value

You can use again the Feymann trick, but now it's more complex: $$\begin{align}\int_{-\infty}^{\infty}\frac{\ln\left(\left|ax+b\right|\right)}{\sqrt{2\pi}}e^{-\frac{x^{2}}{2}}dx=&\frac{\ln\left(\left|a\right|\right)}{\sqrt{2\pi}}\int_{-\infty}^{\infty}e^{-\frac{x^{2}}{2}}dx+\int_{-\infty}^{\infty}\frac{\ln\left(\left|x+\frac{b}{a}\right|\right)}{\sqrt{2\pi}}e^{-\frac{x^{2}}{2}}dx\\ =&\ln\left(\left|a\right|\right)+\int_{-\infty}^{\infty}\frac{\ln\left(\left|x+\frac{b}{a}\right|\right)}{\sqrt{2\pi}}e^{-\frac{x^{2}}{2}}dx\end{align}$$

Let $$G\left(s\right):=\int_{-\infty}^{\infty}\frac{\ln\left(\left|x+s\right|\right)}{\sqrt{2\pi}}e^{-\frac{x^{2}}{2}}dx$$ $$g\left(s\right):=G'(s)=\frac{1}{\sqrt{2\pi}}\text{P.V.}\int_{-\infty}^{\infty}\frac{e^{-\frac{x^{2}}{2}}}{x+s}dx=\sqrt{2}F\left(\frac{s}{\sqrt{2}}\right)$$

So $$G(s)=\frac{s^2}{2}{}_2F_2\left(\left.{1,1\atop\frac{3}{2},2}\right|-\frac{s^2}{2}\right)+c_0$$ $c_0$ is a constant and is determined by imposing the passage for $0$: $$c_0=\int_{-\infty}^{\infty}\frac{\ln\left(\left|x\right|\right)}{\sqrt{2\pi}}e^{-\frac{x^{2}}{2}}dx=-\frac{\ln\left(2\right)+\gamma}{2}$$

So

$$\color{blue}{\int_{-\infty}^{\infty}\frac{\ln\left(\left|ax+b\right|\right)}{\sqrt{2\pi}}e^{-\frac{x^{2}}{2}}dx=\ln(|a|)+\frac{b^2}{2a^2}\cdot{}_2F_2\left(\left.{1,1\atop\frac{3}{2},2}\right|-\frac{b^2}{2a^2}\right)-\frac{\ln\left(2\right)+\gamma}{2}}$$


P.S.: To check the correctness of my formula I leave you two Wolfram links: in the first the one with the integral, in the second the one with the solution (you can change the command and put random values, in this case I put $a=6.2$ and $b=- 2.3$)


Update

In answer to the question if the hypergeometric function can be simplied the answer is no. At most I can give you a numerical method to calculate it yourself:

$${}_2F_2\left(\left.{1,1\atop\frac{3}{2},2}\right|-\frac{z^2}{2}\right)=2\sum_{n=0}^{\infty}\left(-1\right)^{n}\frac{2^{n}n!}{\left(2n+2\right)!}z^{2n}$$